LSAT and Law School Admissions Forum

Get expert LSAT preparation and law school admissions advice from PowerScore Test Preparation.

 Jon Denning
PowerScore Staff
  • PowerScore Staff
  • Posts: 904
  • Joined: Apr 11, 2011
|
#37222
The third question is a Local question with the following condition:

..... U is in the 4th spot

We know from our setup that there are two Us, J (final rule) and either K or F (since G and H are always Rs; reread the setup post if this is unclear).

Since F cannot be in 4 though, as that would force K into 3 where G already is, we have just two options here: J with U in 4, or K with U in 4.

Let's consider both:

..... J in 4: K F G J H
..... K in 4: H J G K F

The question then asks "any of the following could be true EXCEPT:," meaning the correct answer CANNOT be true. Now all we have to do is find an answer that violates both orders given above:

Answer choice (A) could be true because we can have F in 2 (top order)

Answer choice (B) could be true because we can have F in 5 (bottom order)

Answer choice (C) CANNOT be true because only J or K can be 4th. So this answer is correct. (Another way to see this issue is that H is always an R, so if 4 is a U then H is immediately ruled out)

Answer choice (D) could be true because we can have H in 5 (top order)

Answer choice (E) could be true because we can have J in 4 (top order)

Get the most out of your LSAT Prep Plus subscription.

Analyze and track your performance with our Testing and Analytics Package.